[Math] Wieferich’s criterion for Fermat’s Last Theorem

diophantine equationselementary-number-theoryproof-verification

I have found the following way to prove some(Wieferich's) criterion for Fermat's Last Theorem and am wondering what would be wrong. My point of doubt is calculation of the Fermat-quotients of $y,z$ being $-1$, since I found these rules on Wikipedia.

Also, should I split this in parts? I can imagine people don't feel like going through too much text.

Anyway, have fun!

Theorem:

Let:

$ \quad \quad \quad \quad p$ be an odd prime,

$ \quad \quad \quad \quad \gcd(x,y,z) = 1$,

$ \quad \quad \quad \quad xyz \not \equiv 0 \pmod p$

If:

$ \quad \quad \quad \quad x^p = y^p + z^p$,

then $p$ is Wieferich-prime.

Proof:

Consider the following congruence:

$ \quad \quad \quad \quad (x^n – y^n)/(x – y) \equiv nx^{n – 1} \pmod {x – y}$

which we can prove by induction on $n$ and in which we divide first.

Let $n = p$.

Since:

$ \quad \quad \quad \quad \gcd(x – y,(x^p – y^p)/(x – y))$

$ \quad \quad \quad \quad = \gcd(x – y,px^{p – 1})$

$ \quad \quad \quad \quad = \gcd(x – y,p)$

$ \quad \quad \quad \quad = \gcd(x – z,p)$

$ \quad \quad \quad \quad = 1$,

it follows that:

$ \quad \quad \quad \quad x – y = r^p$,

$ \quad \quad \quad \quad (x^p – y^p)/(x – y) = s^p$,

$ \quad \quad \quad \quad x – z = t^p$,

$ \quad \quad \quad \quad (x^p – z^p)/(x – z) = u^p$,

$ \quad \quad \quad \quad rs = z$,

$ \quad \quad \quad \quad tu = y$,

for some $r,s,t,u$ with $\gcd(r,s) = \gcd(t,u) = 1$.

The following also holds for $x – z,t,u$:

$ \quad \quad \quad \quad s \equiv 1 \pmod p \implies s^p \equiv 1 \pmod {p^2}$

Now let:

$ \quad \quad \quad \quad s^p = px^{p – 1} \pmod {x – y}$

$ \quad \quad \quad \quad \implies s^p = px^{p – 1} + ar^p \equiv 1 \pmod {p^2}$, for some $a$

$ \quad \quad \quad \quad \implies s \equiv ar \equiv 1 \pmod p \implies s^p \equiv (ar)^p \equiv 1 \pmod {p^2}$

$ \quad \quad \quad \quad \implies ar^p \equiv 1/a^{p – 1} \pmod {p^2}$

$ \quad \quad \quad \quad \implies s^p = px^{p – 1} + ar^p \equiv px^{p – 1} + 1/a^{p – 1} \equiv 1 \pmod {p^2}$

$ \quad \quad \quad \quad \implies px^{p – 1} \equiv 1 – 1/a^{p – 1} \pmod {p^2}$

$ \quad \quad \quad \quad \implies p(ax)^{p – 1} \equiv a^{p – 1} – 1 \pmod {p^2}$

$ \quad \quad \quad \quad \implies q_p(a) \equiv 1 \pmod p$,

where $q_p(a)$ denotes the Fermat-quotient for $a$ modulo $p$.

So it follows that:

$ \quad \quad \quad \quad q_p(r) \equiv q_p(1/a) \equiv -q_p(a) \equiv -1 \pmod p$

Because of $q_p(s) \equiv 0 \pmod p$:

$ \quad \quad \quad \quad q_p(z) \equiv q_p(rs) \equiv q_p(r) + q_p(s) \equiv -1 + 0 \equiv -1 \pmod p$

Since the same holds for $x – z,t,u$, we now have:

$ \quad \quad \quad \quad q_p(y) \equiv q_p(z) \equiv -1 \pmod p$

From which it follows that:

$ \quad \quad \quad \quad y^{p – 1} \equiv 1 – p \pmod {p^2}$

$ \quad \quad \quad \quad z^{p – 1} \equiv 1 – p \pmod {p^2}$

$ \quad \quad \quad \quad \implies y^p \equiv y(1 – p) \pmod { p^2 }$

$ \quad \quad \quad \quad \implies z^p \equiv z(1 – p) \pmod { p^2 }$

We also note:

$ \quad \quad \quad \quad y^p \equiv (tu)^p \equiv t^p \equiv x – z \pmod {p^2}$

$ \quad \quad \quad \quad z^p \equiv (rs)^p \equiv r^p \equiv x – y \pmod {p^2}$

So we can set:

$ \quad \quad \quad \quad y(1 – p) \equiv x – z \pmod {p^2}$

$ \quad \quad \quad \quad z(1 – p) \equiv x – y \pmod {p^2}$

$ \quad \quad \quad \quad \implies (x – z)/y \equiv (x – y)/z \implies z(x – z) \equiv y(x – y) \pmod {p^2}$

$ \quad \quad \quad \quad \implies y^2 – z^2 \equiv (y + z)(y – z) \equiv x(y – z) \pmod {p^2}$

So either:

$ \quad \quad \quad \quad \implies x \equiv y + z \pmod {p^2}$

or:

$ \quad \quad \quad \quad \implies p | y – z$

Suppose $x \equiv y + z \pmod {p^2}$:

$ \quad \quad \quad \quad \implies (x – y)^p \equiv r^{p^2} \equiv r^p \equiv x – y \implies z^p \equiv z \pmod {p^2}$, contradicting:

$ \quad \quad \quad \quad z^p \equiv z(1 – p) \pmod { p^2 }$

So now we know $y \equiv z \pmod {p}$

But then:

$ \quad \quad \quad \quad y^p \equiv z^p \pmod {p^2}$

$ \quad \quad \quad \quad \implies x^p \equiv y^p + z^p \equiv 2z^p \pmod {p^2}$

Also:

$ \quad \quad \quad \quad x \equiv y + z \implies x \equiv z + z \equiv 2z \pmod p$

$ \quad \quad \quad \quad \implies x^p \equiv (2z)^p \pmod {p^2}$

We conclude:

$ \quad \quad \quad \quad x^p \equiv (2z)^p \equiv 2z^p \pmod {p^2}$

$ \quad \quad \quad \quad \implies (2z)^p – 2z^p \equiv 0 \pmod {p^2}$

$ \quad \quad \quad \quad \implies z^p(2^p – 2) \equiv 0 \pmod {p^2}$,

from which we can see $p$ must be a Wieferich-prime.

Best Answer

A mistake they made is assuming $q_p(s) \equiv q_p(u) \equiv 0 \pmod p$, since $s^{p - 1} \equiv 1/s \pmod {p^2}$.

Related Question